Approximation of $int_t^{infty}e^{-delta(s-t)}omega(s)text{ds}$, if just $omega(t)$,…,$omega^{(n)}(t)$ are...












4












$begingroup$


Consider the following integral: $$int_t^{infty}e^{-delta(s-t)}omega(s)text{ds},$$ where $tgeq 0$ represents the current time, $delta>0$ is a fixed parameter representing the exponential decay rate, and $omega:mathbb{R}_{geq 0}rightarrow mathbb{R}$ is a given smooth bounded function, whose infinite derivatives are also bounded and with none derivative null.



I need to approximate the value of this integral with the restriction that I only have information about the actual $n$-first derivatives $omega(t)$, $dot{omega}(t)$,... ,$omega^{(n)}(t)$, with $ngeq 0$.



Recursive integration by parts leads to
$$int_t^{infty}e^{-delta(s-t)}omega(s)text{ds} = -frac{1}{delta}omega(t) + frac{1}{delta^2}dot{omega}(t)pm...+frac{(-1)^{n+1}}{delta^{n+1}}omega^{(n)}(t)+mathcal{O}_n,$$ where $$mathcal{O}_n=frac{1}{delta^{n+2}}int_t^{infty}e^{-delta(s-t)}omega^{(n+1)}(s)text{ds}.$$



What happens is the following:



Consider a fixed big $delta$ (say $deltagg1$). Then the residual $mathcal{O}_n$ is expected to be small as $1/delta^{n+2}$ is continuously decreasing with respect to $n$. This indicates that, in this case, the integral can be well-approximated by neglecting $mathcal{O}_n$ and, furthermore, the approximation may work better if we have more information about the actual derivatives, i.e. if we can made $n$ arbitrarily large.



However, for a fixed small $delta$ (say $deltall 1$), then the residual is continuously increasing with respect to $n$. This indicates that the approximation does not work as $mathcal{O}_n$ is expected to be big. Furthermore, the approximation may work even worse if we have more information about the actual derivatives!, i.e. making $n$ arbitrarily large.



Could anyone provide an alternative analysis explaining this issue?. Clearly, $deltagg 1$ is needed in order to approximate the integral with this expansion. But, for me is strange because it is just the exponential decay-rate... So why it should be bigger than 1 and not any other number?










share|cite|improve this question











$endgroup$








  • 1




    $begingroup$
    I wonder what $n to infty$ is actually supposed to mean here. The way it is written it seems that $n$ depends on $omega$, so to let $n to infty$, one has to vary $omega$, but then one cannot make much non-trivial comparisons between the integrals for different $n$ and $omega$ anyway. Please clarify.
    $endgroup$
    – Torsten Schoeneberg
    Dec 22 '18 at 6:31










  • $begingroup$
    Well if $n$ is fixed, the dichotomy $a<1$ versus $a>1$ disappears. What you have is that the error term $mathcal {O}_n$ decreases as $a$ increases, continuously depending on $a$, without anything special happening at $a=1$; which makes perfect sense.
    $endgroup$
    – Torsten Schoeneberg
    Dec 24 '18 at 17:34










  • $begingroup$
    Thank you very much for your answers. You have helped me to clarify this question. I have rewritten it again in order to avoid confusions. In any case, even for the case of $n$ fixed (and $delta$ unfixed), it does not make perfect sense for me. I am asking for an alternative analysis that clarifies this behavior. Thanks!
    $endgroup$
    – Alberto Castillo
    Jan 14 at 13:35






  • 1




    $begingroup$
    I suggest finding a more descriptive title for your question.
    $endgroup$
    – Viktor Glombik
    Jan 14 at 13:40










  • $begingroup$
    Remark that $mathcal{O}_n$ depends not only on $delta$ and $n$, but also on $omega^{(n+1)}$ and $s$. It might very well be that that factor in the integral "cancels out" the effect of the $delta^{-n-2}$. Maybe try some easy examples (say $omega(t) = sin(t)$ and $sin^2(t)$) and see what actually happens in those error terms.
    $endgroup$
    – Torsten Schoeneberg
    Jan 14 at 18:05


















4












$begingroup$


Consider the following integral: $$int_t^{infty}e^{-delta(s-t)}omega(s)text{ds},$$ where $tgeq 0$ represents the current time, $delta>0$ is a fixed parameter representing the exponential decay rate, and $omega:mathbb{R}_{geq 0}rightarrow mathbb{R}$ is a given smooth bounded function, whose infinite derivatives are also bounded and with none derivative null.



I need to approximate the value of this integral with the restriction that I only have information about the actual $n$-first derivatives $omega(t)$, $dot{omega}(t)$,... ,$omega^{(n)}(t)$, with $ngeq 0$.



Recursive integration by parts leads to
$$int_t^{infty}e^{-delta(s-t)}omega(s)text{ds} = -frac{1}{delta}omega(t) + frac{1}{delta^2}dot{omega}(t)pm...+frac{(-1)^{n+1}}{delta^{n+1}}omega^{(n)}(t)+mathcal{O}_n,$$ where $$mathcal{O}_n=frac{1}{delta^{n+2}}int_t^{infty}e^{-delta(s-t)}omega^{(n+1)}(s)text{ds}.$$



What happens is the following:



Consider a fixed big $delta$ (say $deltagg1$). Then the residual $mathcal{O}_n$ is expected to be small as $1/delta^{n+2}$ is continuously decreasing with respect to $n$. This indicates that, in this case, the integral can be well-approximated by neglecting $mathcal{O}_n$ and, furthermore, the approximation may work better if we have more information about the actual derivatives, i.e. if we can made $n$ arbitrarily large.



However, for a fixed small $delta$ (say $deltall 1$), then the residual is continuously increasing with respect to $n$. This indicates that the approximation does not work as $mathcal{O}_n$ is expected to be big. Furthermore, the approximation may work even worse if we have more information about the actual derivatives!, i.e. making $n$ arbitrarily large.



Could anyone provide an alternative analysis explaining this issue?. Clearly, $deltagg 1$ is needed in order to approximate the integral with this expansion. But, for me is strange because it is just the exponential decay-rate... So why it should be bigger than 1 and not any other number?










share|cite|improve this question











$endgroup$








  • 1




    $begingroup$
    I wonder what $n to infty$ is actually supposed to mean here. The way it is written it seems that $n$ depends on $omega$, so to let $n to infty$, one has to vary $omega$, but then one cannot make much non-trivial comparisons between the integrals for different $n$ and $omega$ anyway. Please clarify.
    $endgroup$
    – Torsten Schoeneberg
    Dec 22 '18 at 6:31










  • $begingroup$
    Well if $n$ is fixed, the dichotomy $a<1$ versus $a>1$ disappears. What you have is that the error term $mathcal {O}_n$ decreases as $a$ increases, continuously depending on $a$, without anything special happening at $a=1$; which makes perfect sense.
    $endgroup$
    – Torsten Schoeneberg
    Dec 24 '18 at 17:34










  • $begingroup$
    Thank you very much for your answers. You have helped me to clarify this question. I have rewritten it again in order to avoid confusions. In any case, even for the case of $n$ fixed (and $delta$ unfixed), it does not make perfect sense for me. I am asking for an alternative analysis that clarifies this behavior. Thanks!
    $endgroup$
    – Alberto Castillo
    Jan 14 at 13:35






  • 1




    $begingroup$
    I suggest finding a more descriptive title for your question.
    $endgroup$
    – Viktor Glombik
    Jan 14 at 13:40










  • $begingroup$
    Remark that $mathcal{O}_n$ depends not only on $delta$ and $n$, but also on $omega^{(n+1)}$ and $s$. It might very well be that that factor in the integral "cancels out" the effect of the $delta^{-n-2}$. Maybe try some easy examples (say $omega(t) = sin(t)$ and $sin^2(t)$) and see what actually happens in those error terms.
    $endgroup$
    – Torsten Schoeneberg
    Jan 14 at 18:05
















4












4








4





$begingroup$


Consider the following integral: $$int_t^{infty}e^{-delta(s-t)}omega(s)text{ds},$$ where $tgeq 0$ represents the current time, $delta>0$ is a fixed parameter representing the exponential decay rate, and $omega:mathbb{R}_{geq 0}rightarrow mathbb{R}$ is a given smooth bounded function, whose infinite derivatives are also bounded and with none derivative null.



I need to approximate the value of this integral with the restriction that I only have information about the actual $n$-first derivatives $omega(t)$, $dot{omega}(t)$,... ,$omega^{(n)}(t)$, with $ngeq 0$.



Recursive integration by parts leads to
$$int_t^{infty}e^{-delta(s-t)}omega(s)text{ds} = -frac{1}{delta}omega(t) + frac{1}{delta^2}dot{omega}(t)pm...+frac{(-1)^{n+1}}{delta^{n+1}}omega^{(n)}(t)+mathcal{O}_n,$$ where $$mathcal{O}_n=frac{1}{delta^{n+2}}int_t^{infty}e^{-delta(s-t)}omega^{(n+1)}(s)text{ds}.$$



What happens is the following:



Consider a fixed big $delta$ (say $deltagg1$). Then the residual $mathcal{O}_n$ is expected to be small as $1/delta^{n+2}$ is continuously decreasing with respect to $n$. This indicates that, in this case, the integral can be well-approximated by neglecting $mathcal{O}_n$ and, furthermore, the approximation may work better if we have more information about the actual derivatives, i.e. if we can made $n$ arbitrarily large.



However, for a fixed small $delta$ (say $deltall 1$), then the residual is continuously increasing with respect to $n$. This indicates that the approximation does not work as $mathcal{O}_n$ is expected to be big. Furthermore, the approximation may work even worse if we have more information about the actual derivatives!, i.e. making $n$ arbitrarily large.



Could anyone provide an alternative analysis explaining this issue?. Clearly, $deltagg 1$ is needed in order to approximate the integral with this expansion. But, for me is strange because it is just the exponential decay-rate... So why it should be bigger than 1 and not any other number?










share|cite|improve this question











$endgroup$




Consider the following integral: $$int_t^{infty}e^{-delta(s-t)}omega(s)text{ds},$$ where $tgeq 0$ represents the current time, $delta>0$ is a fixed parameter representing the exponential decay rate, and $omega:mathbb{R}_{geq 0}rightarrow mathbb{R}$ is a given smooth bounded function, whose infinite derivatives are also bounded and with none derivative null.



I need to approximate the value of this integral with the restriction that I only have information about the actual $n$-first derivatives $omega(t)$, $dot{omega}(t)$,... ,$omega^{(n)}(t)$, with $ngeq 0$.



Recursive integration by parts leads to
$$int_t^{infty}e^{-delta(s-t)}omega(s)text{ds} = -frac{1}{delta}omega(t) + frac{1}{delta^2}dot{omega}(t)pm...+frac{(-1)^{n+1}}{delta^{n+1}}omega^{(n)}(t)+mathcal{O}_n,$$ where $$mathcal{O}_n=frac{1}{delta^{n+2}}int_t^{infty}e^{-delta(s-t)}omega^{(n+1)}(s)text{ds}.$$



What happens is the following:



Consider a fixed big $delta$ (say $deltagg1$). Then the residual $mathcal{O}_n$ is expected to be small as $1/delta^{n+2}$ is continuously decreasing with respect to $n$. This indicates that, in this case, the integral can be well-approximated by neglecting $mathcal{O}_n$ and, furthermore, the approximation may work better if we have more information about the actual derivatives, i.e. if we can made $n$ arbitrarily large.



However, for a fixed small $delta$ (say $deltall 1$), then the residual is continuously increasing with respect to $n$. This indicates that the approximation does not work as $mathcal{O}_n$ is expected to be big. Furthermore, the approximation may work even worse if we have more information about the actual derivatives!, i.e. making $n$ arbitrarily large.



Could anyone provide an alternative analysis explaining this issue?. Clearly, $deltagg 1$ is needed in order to approximate the integral with this expansion. But, for me is strange because it is just the exponential decay-rate... So why it should be bigger than 1 and not any other number?







calculus integration






share|cite|improve this question















share|cite|improve this question













share|cite|improve this question




share|cite|improve this question








edited Jan 17 at 8:09







Alberto Castillo

















asked Dec 21 '18 at 16:47









Alberto CastilloAlberto Castillo

212




212








  • 1




    $begingroup$
    I wonder what $n to infty$ is actually supposed to mean here. The way it is written it seems that $n$ depends on $omega$, so to let $n to infty$, one has to vary $omega$, but then one cannot make much non-trivial comparisons between the integrals for different $n$ and $omega$ anyway. Please clarify.
    $endgroup$
    – Torsten Schoeneberg
    Dec 22 '18 at 6:31










  • $begingroup$
    Well if $n$ is fixed, the dichotomy $a<1$ versus $a>1$ disappears. What you have is that the error term $mathcal {O}_n$ decreases as $a$ increases, continuously depending on $a$, without anything special happening at $a=1$; which makes perfect sense.
    $endgroup$
    – Torsten Schoeneberg
    Dec 24 '18 at 17:34










  • $begingroup$
    Thank you very much for your answers. You have helped me to clarify this question. I have rewritten it again in order to avoid confusions. In any case, even for the case of $n$ fixed (and $delta$ unfixed), it does not make perfect sense for me. I am asking for an alternative analysis that clarifies this behavior. Thanks!
    $endgroup$
    – Alberto Castillo
    Jan 14 at 13:35






  • 1




    $begingroup$
    I suggest finding a more descriptive title for your question.
    $endgroup$
    – Viktor Glombik
    Jan 14 at 13:40










  • $begingroup$
    Remark that $mathcal{O}_n$ depends not only on $delta$ and $n$, but also on $omega^{(n+1)}$ and $s$. It might very well be that that factor in the integral "cancels out" the effect of the $delta^{-n-2}$. Maybe try some easy examples (say $omega(t) = sin(t)$ and $sin^2(t)$) and see what actually happens in those error terms.
    $endgroup$
    – Torsten Schoeneberg
    Jan 14 at 18:05
















  • 1




    $begingroup$
    I wonder what $n to infty$ is actually supposed to mean here. The way it is written it seems that $n$ depends on $omega$, so to let $n to infty$, one has to vary $omega$, but then one cannot make much non-trivial comparisons between the integrals for different $n$ and $omega$ anyway. Please clarify.
    $endgroup$
    – Torsten Schoeneberg
    Dec 22 '18 at 6:31










  • $begingroup$
    Well if $n$ is fixed, the dichotomy $a<1$ versus $a>1$ disappears. What you have is that the error term $mathcal {O}_n$ decreases as $a$ increases, continuously depending on $a$, without anything special happening at $a=1$; which makes perfect sense.
    $endgroup$
    – Torsten Schoeneberg
    Dec 24 '18 at 17:34










  • $begingroup$
    Thank you very much for your answers. You have helped me to clarify this question. I have rewritten it again in order to avoid confusions. In any case, even for the case of $n$ fixed (and $delta$ unfixed), it does not make perfect sense for me. I am asking for an alternative analysis that clarifies this behavior. Thanks!
    $endgroup$
    – Alberto Castillo
    Jan 14 at 13:35






  • 1




    $begingroup$
    I suggest finding a more descriptive title for your question.
    $endgroup$
    – Viktor Glombik
    Jan 14 at 13:40










  • $begingroup$
    Remark that $mathcal{O}_n$ depends not only on $delta$ and $n$, but also on $omega^{(n+1)}$ and $s$. It might very well be that that factor in the integral "cancels out" the effect of the $delta^{-n-2}$. Maybe try some easy examples (say $omega(t) = sin(t)$ and $sin^2(t)$) and see what actually happens in those error terms.
    $endgroup$
    – Torsten Schoeneberg
    Jan 14 at 18:05










1




1




$begingroup$
I wonder what $n to infty$ is actually supposed to mean here. The way it is written it seems that $n$ depends on $omega$, so to let $n to infty$, one has to vary $omega$, but then one cannot make much non-trivial comparisons between the integrals for different $n$ and $omega$ anyway. Please clarify.
$endgroup$
– Torsten Schoeneberg
Dec 22 '18 at 6:31




$begingroup$
I wonder what $n to infty$ is actually supposed to mean here. The way it is written it seems that $n$ depends on $omega$, so to let $n to infty$, one has to vary $omega$, but then one cannot make much non-trivial comparisons between the integrals for different $n$ and $omega$ anyway. Please clarify.
$endgroup$
– Torsten Schoeneberg
Dec 22 '18 at 6:31












$begingroup$
Well if $n$ is fixed, the dichotomy $a<1$ versus $a>1$ disappears. What you have is that the error term $mathcal {O}_n$ decreases as $a$ increases, continuously depending on $a$, without anything special happening at $a=1$; which makes perfect sense.
$endgroup$
– Torsten Schoeneberg
Dec 24 '18 at 17:34




$begingroup$
Well if $n$ is fixed, the dichotomy $a<1$ versus $a>1$ disappears. What you have is that the error term $mathcal {O}_n$ decreases as $a$ increases, continuously depending on $a$, without anything special happening at $a=1$; which makes perfect sense.
$endgroup$
– Torsten Schoeneberg
Dec 24 '18 at 17:34












$begingroup$
Thank you very much for your answers. You have helped me to clarify this question. I have rewritten it again in order to avoid confusions. In any case, even for the case of $n$ fixed (and $delta$ unfixed), it does not make perfect sense for me. I am asking for an alternative analysis that clarifies this behavior. Thanks!
$endgroup$
– Alberto Castillo
Jan 14 at 13:35




$begingroup$
Thank you very much for your answers. You have helped me to clarify this question. I have rewritten it again in order to avoid confusions. In any case, even for the case of $n$ fixed (and $delta$ unfixed), it does not make perfect sense for me. I am asking for an alternative analysis that clarifies this behavior. Thanks!
$endgroup$
– Alberto Castillo
Jan 14 at 13:35




1




1




$begingroup$
I suggest finding a more descriptive title for your question.
$endgroup$
– Viktor Glombik
Jan 14 at 13:40




$begingroup$
I suggest finding a more descriptive title for your question.
$endgroup$
– Viktor Glombik
Jan 14 at 13:40












$begingroup$
Remark that $mathcal{O}_n$ depends not only on $delta$ and $n$, but also on $omega^{(n+1)}$ and $s$. It might very well be that that factor in the integral "cancels out" the effect of the $delta^{-n-2}$. Maybe try some easy examples (say $omega(t) = sin(t)$ and $sin^2(t)$) and see what actually happens in those error terms.
$endgroup$
– Torsten Schoeneberg
Jan 14 at 18:05






$begingroup$
Remark that $mathcal{O}_n$ depends not only on $delta$ and $n$, but also on $omega^{(n+1)}$ and $s$. It might very well be that that factor in the integral "cancels out" the effect of the $delta^{-n-2}$. Maybe try some easy examples (say $omega(t) = sin(t)$ and $sin^2(t)$) and see what actually happens in those error terms.
$endgroup$
– Torsten Schoeneberg
Jan 14 at 18:05












0






active

oldest

votes











Your Answer





StackExchange.ifUsing("editor", function () {
return StackExchange.using("mathjaxEditing", function () {
StackExchange.MarkdownEditor.creationCallbacks.add(function (editor, postfix) {
StackExchange.mathjaxEditing.prepareWmdForMathJax(editor, postfix, [["$", "$"], ["\\(","\\)"]]);
});
});
}, "mathjax-editing");

StackExchange.ready(function() {
var channelOptions = {
tags: "".split(" "),
id: "69"
};
initTagRenderer("".split(" "), "".split(" "), channelOptions);

StackExchange.using("externalEditor", function() {
// Have to fire editor after snippets, if snippets enabled
if (StackExchange.settings.snippets.snippetsEnabled) {
StackExchange.using("snippets", function() {
createEditor();
});
}
else {
createEditor();
}
});

function createEditor() {
StackExchange.prepareEditor({
heartbeatType: 'answer',
autoActivateHeartbeat: false,
convertImagesToLinks: true,
noModals: true,
showLowRepImageUploadWarning: true,
reputationToPostImages: 10,
bindNavPrevention: true,
postfix: "",
imageUploader: {
brandingHtml: "Powered by u003ca class="icon-imgur-white" href="https://imgur.com/"u003eu003c/au003e",
contentPolicyHtml: "User contributions licensed under u003ca href="https://creativecommons.org/licenses/by-sa/3.0/"u003ecc by-sa 3.0 with attribution requiredu003c/au003e u003ca href="https://stackoverflow.com/legal/content-policy"u003e(content policy)u003c/au003e",
allowUrls: true
},
noCode: true, onDemand: true,
discardSelector: ".discard-answer"
,immediatelyShowMarkdownHelp:true
});


}
});














draft saved

draft discarded


















StackExchange.ready(
function () {
StackExchange.openid.initPostLogin('.new-post-login', 'https%3a%2f%2fmath.stackexchange.com%2fquestions%2f3048686%2fapproximation-of-int-t-inftye-deltas-t-omegas-textds-if-just%23new-answer', 'question_page');
}
);

Post as a guest















Required, but never shown

























0






active

oldest

votes








0






active

oldest

votes









active

oldest

votes






active

oldest

votes
















draft saved

draft discarded




















































Thanks for contributing an answer to Mathematics Stack Exchange!


  • Please be sure to answer the question. Provide details and share your research!

But avoid



  • Asking for help, clarification, or responding to other answers.

  • Making statements based on opinion; back them up with references or personal experience.


Use MathJax to format equations. MathJax reference.


To learn more, see our tips on writing great answers.




draft saved


draft discarded














StackExchange.ready(
function () {
StackExchange.openid.initPostLogin('.new-post-login', 'https%3a%2f%2fmath.stackexchange.com%2fquestions%2f3048686%2fapproximation-of-int-t-inftye-deltas-t-omegas-textds-if-just%23new-answer', 'question_page');
}
);

Post as a guest















Required, but never shown





















































Required, but never shown














Required, but never shown












Required, but never shown







Required, but never shown

































Required, but never shown














Required, but never shown












Required, but never shown







Required, but never shown







Popular posts from this blog

Mario Kart Wii

What does “Dominus providebit” mean?

Antonio Litta Visconti Arese